LSAT and Law School Admissions Forum

Get expert LSAT preparation and law school admissions advice from PowerScore Test Preparation.

 Administrator
PowerScore Staff
  • PowerScore Staff
  • Posts: 8929
  • Joined: Feb 02, 2011
|
#91287
Complete Question Explanation

The correct answer choice is (C).

Answer choice (A):

Answer choice (B):

Answer choice (C): This is the correct answer choice.

Answer choice (D):

Answer choice (E):

This explanation is still in progress. Please post any questions below!
User avatar
 rdee81
  • Posts: 18
  • Joined: Aug 23, 2021
|
#95049
I got this one right on my own by going by my initial set up which is HMJLKF. Was this the right approach?
 Rachael Wilkenfeld
PowerScore Staff
  • PowerScore Staff
  • Posts: 1392
  • Joined: Dec 15, 2011
|
#95082
Hi rdee,

I'm not sure what you mean by your initial set-up. That is the correct order if H is in 1. However it's not the only order the variables can occur. I would do this more systematically. How many variables must be after L? 2. That means L plus the two after can only go as late as 4, 5, and 6. That would mean at most there are three variables before L.

Hope that helps!

Get the most out of your LSAT Prep Plus subscription.

Analyze and track your performance with our Testing and Analytics Package.